LSAT and Law School Admissions Forum

Get expert LSAT preparation and law school admissions advice from PowerScore Test Preparation.

 Administrator
PowerScore Staff
  • PowerScore Staff
  • Posts: 8917
  • Joined: Feb 02, 2011
|
#26254
Complete Question Explanation
(The complete setup for this game can be found here: lsat/viewtopic.php?t=10917)

The correct answer choice is (D)

The condition in this question stem is sequencing in nature (F :longline: H), requiring us to modify our sequencing diagram as follows:
June15_game_3_#17_diagram_1.png
It is clear from this diagram that F :longline: L, which triggers the sufficient condition in the last rule. Consequently, we need to ensure that L is scheduled on Saturday:
June15_game_3_#17_diagram_2.png
To answer this Must Be True question as quickly as possible, look for an answer choice involving L. Its placement on Saturday is the only inference we can make here, and is likely to play a key role in the wording of the correct answer choice.

Answer choice (A): This answer choice is incorrect, because the relationship between H and K is unknown.

Answer choice (B): This answer choice is incorrect, because the relationship between H and Z is unknown. As long as G is on duty on Saturday, Z could be scheduled earlier than H.

Answer choice (C): This answer choice is incorrect, because the relationship between K and M is unknown.

Answer choice (D): This is the correct answer choice. If L is scheduled on Saturday, then all other variables, except for G and Z, must be scheduled earlier than L.

Answer choice (E): This answer choice is incorrect, because the relationship between M and Z is unknown.
You do not have the required permissions to view the files attached to this post.

Get the most out of your LSAT Prep Plus subscription.

Analyze and track your performance with our Testing and Analytics Package.